Difference between revisions of "1975 AHSME Problems/Problem 20"

(Created page with "Let <math>BM=CM=x</math>. Then, by Stewart's Theorem, we have <cmath>2x^3+18x=16x+64x</cmath> <cmath>\implies x^2+9=40</cmath> <cmath>\implies x=\sqrt{31}\implies BC=\boxed{2\...")
 
 
(3 intermediate revisions by 2 users not shown)
Line 1: Line 1:
 +
==Problem==
 +
In the adjoining figure triangle <math>ABC</math> is such that <math>AB = 4</math> and <math>AC = 8</math>. IF <math>M</math> is the midpoint of <math>BC</math> and <math>AM = 3</math>, what is the length of <math>BC</math>?
 +
 +
<asy>
 +
draw((-4,0)--(4,0)--(-1,4)--cycle);
 +
draw((-1, 4)--(0, 0.00001));
 +
label("B", (-4,0), S);
 +
label("C", (4,0), S);
 +
label("A", (-1, 4), N);
 +
label("M", (0, 0.0001), S);
 +
</asy>
 +
 +
<math>
 +
\textbf{(A)}\ 2\sqrt{26} \qquad \textbf{(B)}\ 2\sqrt{31} \qquad \textbf{(C)}\ 9 \qquad \textbf{(D)}\ 4 + 2\sqrt{13}\\ \textbf{(E)}\ \text{not enough information given to solve the problem}
 +
</math>
 +
 +
==Solution==
 
Let <math>BM=CM=x</math>. Then, by Stewart's Theorem, we have
 
Let <math>BM=CM=x</math>. Then, by Stewart's Theorem, we have
 
<cmath>2x^3+18x=16x+64x</cmath>
 
<cmath>2x^3+18x=16x+64x</cmath>
 
<cmath>\implies x^2+9=40</cmath>
 
<cmath>\implies x^2+9=40</cmath>
 
<cmath>\implies x=\sqrt{31}\implies BC=\boxed{2\sqrt{31}}.</cmath>
 
<cmath>\implies x=\sqrt{31}\implies BC=\boxed{2\sqrt{31}}.</cmath>
 +
The answer is <math>\boxed{B}.</math>
 
-brainiacmaniac31
 
-brainiacmaniac31
 +
 +
==See Also==
 +
{{AHSME box|year=1975|num-b=19|num-a=21}}
 +
{{MAA Notice}}

Latest revision as of 17:32, 19 January 2021

Problem

In the adjoining figure triangle $ABC$ is such that $AB = 4$ and $AC = 8$. IF $M$ is the midpoint of $BC$ and $AM = 3$, what is the length of $BC$?

[asy] draw((-4,0)--(4,0)--(-1,4)--cycle); draw((-1, 4)--(0, 0.00001)); label("B", (-4,0), S); label("C", (4,0), S); label("A", (-1, 4), N); label("M", (0, 0.0001), S); [/asy]

$\textbf{(A)}\ 2\sqrt{26} \qquad \textbf{(B)}\ 2\sqrt{31} \qquad \textbf{(C)}\ 9 \qquad \textbf{(D)}\ 4 + 2\sqrt{13}\\ \textbf{(E)}\ \text{not enough information given to solve the problem}$

Solution

Let $BM=CM=x$. Then, by Stewart's Theorem, we have \[2x^3+18x=16x+64x\] \[\implies x^2+9=40\] \[\implies x=\sqrt{31}\implies BC=\boxed{2\sqrt{31}}.\] The answer is $\boxed{B}.$ -brainiacmaniac31

See Also

1975 AHSME (ProblemsAnswer KeyResources)
Preceded by
Problem 19
Followed by
Problem 21
1 2 3 4 5 6 7 8 9 10 11 12 13 14 15 16 17 18 19 20 21 22 23 24 25 26 27 28 29 30
All AHSME Problems and Solutions

The problems on this page are copyrighted by the Mathematical Association of America's American Mathematics Competitions. AMC logo.png